Questions tagged [ds.dynamical-systems]

Dynamics of flows and maps (continuous and discrete time), including infinite-dimensional dynamics, Hamiltonian dynamics, ergodic theory.

Filter by
Sorted by
Tagged with
3 votes
1 answer
238 views

Does Bernoulli imply exponential mixing?

This question comes from this paper where the authors proved that exponential mixing implies Bernoulli. They also mentioned in the introduction that Bernoulli is the strongest ergodic property and ...
Kousaka_Reina's user avatar
1 vote
1 answer
69 views

Number of ergodic transverse measures for geodesic laminations - bounded by the genus?

Consider a geodesic lamination $\Lambda$`of a closed hyperbolic surface $S$ of genus $g$, and take a globally transverse closed curve $I$. The lamination induces a return map $R_{\Lambda}: I \to I$, ...
Alejo García Sassi's user avatar
1 vote
0 answers
94 views

Question about ergodic flows and periodicity

Let $X$ be a compact Haussdorf space, let $\mu$ be a Borel measure on $X$ with $\mathrm{supp}(\mu)=X$ and let $(\phi_s)_{s\in\mathbb R}$ be a one-parameter group of homeomorphisms which is continuous ...
Lau's user avatar
  • 729
1 vote
1 answer
116 views

Does every proximal dynamical system have zero topological entropy?

A dynamical system is proximal if $$\:\forall (x,y) \in X \times X, \: \liminf_{n \rightarrow \infty} d(f^{n}(x),f^{n}(y)) = 0 $$ (where $X$ is a compact metric space with metric $d$). Is it true that ...
Matej Moravik's user avatar
2 votes
0 answers
135 views

Unipotent closure in classical groups

Let $G=\mathrm{SL}_n(\mathbb{R}),\mathrm{Sp}_{2n}(\mathbb{R}),\mathrm{Spin}_n(\mathbb{R})$ be a semi-simple simply connected classical group, $\Gamma\subset G$ a discrete and cocompact subgroup. Then ...
Mathew's user avatar
  • 21
8 votes
4 answers
791 views

Ergodic theory applied to number theory

I am interested in the links between Ergodic Theory and Number Theory. Can anyone give some references for papers to read in this field? Any open problems? Or ideas where it may be applicable in NT?
0 votes
0 answers
49 views

Sufficient conditions for chain recurrent set equal to set of non wandering points

Given a generic diffeomorphism, I know that the set of nonwandering points is contained in the chain recurrent set, but the converse is not always true. Is there some sufficient conditions under which ...
giangian's user avatar
  • 101
3 votes
1 answer
72 views

Rate of convergence for Markov chain in random environment

Let $(\Omega,\mathfrak{F},\mathbb{P})$ be a probability space and $\sigma:\Omega\to\Omega$ be an ergodic, invertible and measure preserving transformation. Consider a family of column stochastic ...
JayP's user avatar
  • 131
5 votes
0 answers
218 views

Is the global solution to this ODE bounded?

Consider $$\dot{\theta_i}=-\sum_{j=1}^nA_{ij}\sin(\theta_i-\theta_j),\ i\in\{1,2,\cdots,n\}$$ where $A_{ij}$ is adjacency matrix of a connected simple graph, and the vector $\theta=[\theta_1,\cdots,\...
tony's user avatar
  • 329
2 votes
1 answer
271 views

When does uniqueness of a stable equilibrium imply it is globally stable?

Given a gradient dynamical system $$\dot x=-\nabla f(x),$$ my question is: (1) If there exists only one equilibrium $x^*$ which is stable (if necessary, this can be changed to stable asymptotically ...
tony's user avatar
  • 329
1 vote
0 answers
74 views

Closed subgroups in Ratner's orbit closure theorem on unipotent flows

Let $G$ be a semisimple (real or $p$-adic) Lie group and $\Gamma$ a discrete and cocompact subgroup of $G$, as in the setting of Ratner's theorems on unipotent flows (see for example here \url{https://...
Zhang's user avatar
  • 81
3 votes
0 answers
48 views

Stability of indefinitely damped mechanical system with diagonal stiffness

I'm trying to find conditions for the asymptotic stability of the following linear system, \begin{equation} \mathbf{I \ddot{x}} + \mathbf{B \dot{x}} + \mathbf{K x} = 0 \end{equation} given the ...
Shivang Rawat's user avatar
0 votes
0 answers
25 views

Coefficients in the series expansion of a central manifold are all zero

I have a system of 4 ODEs, which linearized around the origin gives $$ \begin{align} &\dot{q_1}=a\, q_1\\ &\dot{q_2}=b\,q_2\\ &\dot{q_3}=0\\ &\dot{q_4}=c\,q_4 \end{align} $$ with $a$, $...
F.Mor's user avatar
  • 1
3 votes
1 answer
134 views

Topological amenability of actions - forgetting topology

Let $G$ be a (countable) discrete group and let $X$ be a locally compact Hausdorff space. Assume that $G$ acts on $X$ by homeomorphisms. Recall that the action is (topologically) amenable if there ...
Alcides Buss's user avatar
3 votes
1 answer
162 views

Chain components and posets

Let $(X,f)$ be a topological dynamical system ($f$ continuous, $X$ compact, metric with distance $d$). Let $C\subseteq X^2$ indicate the chain recurrence relation: $$xCy\iff \forall \epsilon>0\ \...
Alessandro Della Corte's user avatar
0 votes
1 answer
153 views

Help in understanding the singular system of linear forms and non escape of mass

I am having some trouble in understanding certain portions of the following paper by KKLM https://link.springer.com/article/10.1007/s11854-017-0033-4 So in proposition 3.1, they proved the estimate ...
User1723's user avatar
  • 307
0 votes
0 answers
55 views

Diophantine-like approximation of dynamical subsystems

For $\alpha\in [0,1)$ irrational we know that there exists a sequences $\{ q_n \}_{n=1}^\infty\subseteq \mathbb{N}$ and $\{ p_n \}_{n=1}^\infty\subseteq \mathbb{Z}$ such that $$ \Big\vert \alpha-\frac{...
Keen-ameteur's user avatar
0 votes
0 answers
33 views

Existence of a minimal, weakly mixing and Lipschitz selfmap?

I am looking for an example of a dynamical system $(M,f)$ such that: $M$ is a metric space; $f:M \to M$ is Lipschitz; $f$ is weakly mixing (that is $f \times f$ is topologically transitive) $f$ is ...
Baguette's user avatar
2 votes
0 answers
62 views

Name for a product of actions / dynamical systems

Suppose $G \curvearrowright X, H \curvearrowright Y$ are group (or monoid) actions, or dynamical systems. Then $X \times Y$ is a $G \times H$-system of the same type in the obvious way by $(g, h) \...
Ville Salo's user avatar
  • 6,337
0 votes
0 answers
88 views

How to show that the map $ R $ here is measure-preserving

Assume that $ (X,\mathcal{B},m,T) $ is a measure-preserving dynamical system, where $ (X,\mathcal{B},m) $ is a probability space, $ \mathcal{B} $ denotes all the measurable sets in $ X $, $ m $ is the ...
Luis Yanka Annalisc's user avatar
5 votes
1 answer
289 views

Weak mixing and entering time

Let $X$ be a compact metric space and $f$ a continuous map from $X$ to $X$. Is it true, that if $f$ is weakly mixing, then the entering time $$N(U,V) = \{n \in \mathbb{N}\mid f^n(U) \cap V \neq \...
Matěj Moravík's user avatar
3 votes
0 answers
155 views

Has this metric been considered anywhere?

I posted this on math stack exchange some 10 days ago, but received no answers (https://math.stackexchange.com/q/4773194/1223994). Let $X$ be a compact metric space and denote by $d$ the metric on $X$....
Marco Farotti's user avatar
1 vote
0 answers
45 views

Does a substitution tiling being FLC depend on starting seed?

I've been trying to understand more on "geometric" substitutions rather than just symbolic ones. As symbolic substitutions always yield FLC tilings, I wanted to know whether a tiling coming ...
Keen-ameteur's user avatar
2 votes
1 answer
112 views

Any theorem shows that flowmap $\phi_{\sum_{i=1}^n a_i f_i(x)}^\tau$ can be approximated by $\phi_{f_{\theta(t)}(x)}^{\tau'}$?

Given a control family $F:=\{f_1,\dotsc,f_n\}$, and $\phi_f^\tau(x)$ is the flowmap of the dynamical system $$ \begin{cases} z'(t)=f(z),\\ z(0)=x, \end{cases} $$ at end time point $\tau$. Suppose $a_i&...
li ang Duan's user avatar
1 vote
1 answer
255 views

Alternate definitions of compact and weak mixing extensions

In Furstenberg's proof of the multiple recurrence theorem in ergodic theory, one makes use of the concept of compact and weak mixing extensions of a measure preserving system. The following definition ...
Nate River's user avatar
  • 4,832
11 votes
1 answer
3k views

Understanding the application of two inequalities?

I am reading the paper "The long-time behaviour of a stochastic SIR epidemic model with distributed delay and multidimensional Levy jumps" by Driss Kiouach and Yassine Sabbar. I have two ...
Math's user avatar
  • 185
1 vote
1 answer
153 views

Gradient descent under the presence of symmetries

Let $M$ be a Riemannian manifold (I'm happy to assume it is Euclidean space) with a function $f: M \to \mathbb R$ and a group of isometries $G$ acting on $M$ and preserving $f$, i.e., $f(gm) = f(m)$ ...
Asvin's user avatar
  • 7,646
13 votes
4 answers
1k views

Continuous dynamical systems and analytic number theory: connections

Are there any connections between continuous dynamical systems and (analytic) number theory?
Albert Loewenstein's user avatar
3 votes
2 answers
386 views

Functional equations based on composition

I have asked this question here (*), but there are no answer. Let $n \in \mathbb N^*$, $\{a_0,\ldots,a_n\} \subset \left] 0,+\infty\right]$. We suppose $Eq : \sum\limits_{k=0}^n a_k f^k(x)=0$ have no ...
Dattier's user avatar
  • 3,801
4 votes
0 answers
107 views

Decidability of whether two polynomial bijections generate a free group

I am wondering about the decidability of the following question: Given two polynomial bijections $f, g$ from the real numbers to the real numbers (with say rational coefficient just to simplify what &...
Sprotte's user avatar
  • 1,065
0 votes
1 answer
39 views

Is the right-hand term of the autonomous dynamic system equivalent to the original system after being multiplied by a constant?

Given two dynamical systems where $f$ is lipschitz for $x$ : $\begin{cases} x'(t)=af(x),\\ x(0)=x_0,\end{cases} t\in[0,\tau]$ and $\begin{cases} z'(t)=f(z),\\ z(0)=x_0,\end{cases} t\in[0,\tau']$, and ...
li ang Duan's user avatar
-1 votes
1 answer
292 views

Is this submonoid of the isometry group on $\Bbb Q_2$ closed to inverses? [closed]

Let $\textrm{aff}(ax+b)$ be the affine group on $\Bbb Z_2^\times$ i.e. the set of linear polynomials over 2-adic numbers with $a\in\Bbb Z_2^\times, b\in\Bbb Z_2$ Now let $X$ be the restriction of its ...
it's a hire car baby's user avatar
0 votes
2 answers
68 views

Is the right-hand term of the dynamic system equivalent to the original system after being multiplied by a constant?

Given two dynamical systems where $f$ is lipschitz for $x$ : $\begin{cases} x'(t)=af(x,t),\\ x(0)=x_0,\end{cases} t\in[0,\tau]$ and $\begin{cases} z'(t)=f(z,t),\\ z(0)=x_0,\end{cases} t\in[0,\tau']$, ...
li ang Duan's user avatar
1 vote
0 answers
63 views

Physical measure of a dynamical system in terms of its density

Let $f$ be a $\mathcal{C}^1$ vector field on a compact subset $M \subset \mathbb{R}^n$. We define a dynamical system by $$\dot{x}(t)=f(x(t))$$ In ergodic theory, the occupation measure is $$\mu_{x, T}(...
NicAG's user avatar
  • 247
4 votes
0 answers
239 views

Dynamical obstruction for a vector field to have a Harmonic divergence

Let $(M,g)$ be an analytic Riemannian manifold and $X$ be an analytic vector field on $M$. Can we always have a volume form $\Omega$ such that $\operatorname{Div}_{\Omega} X$ is a harmonic ...
Ali Taghavi's user avatar
1 vote
0 answers
147 views

The space of ergodic elements of a topological or Lie group

Let $G$ be a compact topological group with normalized Haar measure $\mu$. An element $g\in G$ is an ergodic element if the mapping $L_g:G \to G $ with $x\mapsto gx$ is an ergodic map. The ...
Ali Taghavi's user avatar
1 vote
0 answers
39 views

The boundedness of dynamical systems discretized from Hamiltonian systems

Let $H(p,q) = T(q) + U(p)$ be a Hamiltonian function that defines a Hamiltonian system, i.e., \begin{align} &\frac{dp}{dt} = \frac{\partial H}{\partial q}(p,q) = \frac{dT}{dq},\\ &\frac{dq}{dt}...
Yi_Feng's user avatar
  • 47
3 votes
0 answers
150 views

Local dimension of stationary measures for iterated function systems with an expanding map

Consider the iterated function system (IFS) $X_n$ on $I = [0,1] $generated by the functions $\Phi = \{f_1,f_2,f_3\}$ and the probability vector $P = (p/2,p/2,1-p),$ where: $f_1,f_2: I\to I$, where $...
Matheus Manzatto's user avatar
0 votes
0 answers
55 views

Distortion lemma for composition of (distinct) functions expanding on average

I am trying to describe the following dynamics: Let $(T_{\rho})_{\rho \in [0, 1]}$, $T_{\rho}: [-1, 1] \rightarrow [-1, 1]$ be a family map which satisfies: $\forall \, \rho \in [0, 1], \, \exists \,...
Gabriel B. H. Lisboa's user avatar
7 votes
1 answer
1k views

If the pointwise ergodic theorem holds along all subsequences with nonzero natural density, is the system strong mixing?

Let $\mathbf X := (X, \mathcal S, \mu, T)$ be an ergodic measure preserving system with finite measure such that for every increasing sequence $\{n_k\}$ of natural numbers whose natural density exists ...
Nate River's user avatar
  • 4,832
0 votes
0 answers
50 views

Role of basins of attraction in the Morse decomposition

Let $M$ be a differentiable manifold and $F \in \mathcal{X}(M)$. We define a DS by $$\dot{x}=F(x(t))$$ An ordered collection $\mathcal{M}=\left\{M_{1}, \ldots, M_{l}\right\}$ of compact subsets of ...
NicAG's user avatar
  • 247
1 vote
0 answers
161 views

Rotation number for homeomorphisms of a Lie group other than $S^1$

Let $G$ be a Lie group whose Lie algebra is $\mathfrak{g}$ with exponential map $\exp:\mathfrak{g}\to G$. For what kind of Lie group $G$ the standard process of definition of rotation number ...
Ali Taghavi's user avatar
1 vote
0 answers
172 views

Is the Poincare Birkhoff theorem valid if we change the volume form of the annulus region?

Is the Poincare-Birkhoff theorem valid if we change the volume form of the annulus region? Note: A possible approach could be the following: Is it true to say that the answer is affirmative ...
Ali Taghavi's user avatar
0 votes
0 answers
38 views

Generic non-existence of 1. Integral of continuous DS

Let $M$ be a compact manfiold and $F \in \mathcal{X}(M)$. We define a DS on $M$ by $$\dot{\mathbf{x}}=F(\mathbf{x}(t))$$ In 1 it was shown by Hurley, that a generic diffeomorphism on $M$ does not have ...
NicAG's user avatar
  • 247
5 votes
1 answer
186 views

Bias of DS literature to polynomial ODEs

In the literature on continuous time dynamical system, we generally deal with an open set $U \subset \mathbb{R}^n$ and a vector field $F: U \rightarrow \mathbb{R}^n$ and define a DS by the ODE $$\...
NicAG's user avatar
  • 247
2 votes
3 answers
342 views

Intersection of Fourier analysis (especially on the transform) and group theory, number theory, dynamical systems, etc

I am considering a PhD research topic. I only have a math Bachelor's degree with working experience mostly in teaching and I have been working on a paper. I have deep interest in Fourier Series and ...
3 votes
1 answer
294 views

Equidistribution of the orbit $\{\text{diag}(t^a,t^{-a})\Lambda \}_{t>0}$ for a.e. $\Lambda\in \text{SL}(2,\mathbb R)/\text{SL}(2,\mathbb Z)$

$\DeclareMathOperator\diag{diag}\DeclareMathOperator\SL{SL}$It is well-known that geodesic flow $g_t=\{\diag(e^t,e^{-t}) \}_{t>0}$ acts ergodically (actually mixing) on $\SL(2,\mathbb R)$ (Howe–...
user506835's user avatar
4 votes
0 answers
173 views

References for derivative w.r.t. initial condition of an ODE

Let $b:\mathbb R_+ \times \mathbb R^d \to \mathbb R^d$ be measurable such that for all $n \in \mathbb N$ we have $$ \sup_{t \ge 0} |b(t, 0)| + \sup_{t \ge 0} \sup_{x \in \mathbb R^d} |\nabla^n_x b (t, ...
Akira's user avatar
  • 851
0 votes
0 answers
296 views

Proof that a first integral is not a constant function

Let $U$ be an (open) set in $\mathbb{R}^n$. And we are given a set of $m$ basis functions $$B=\{\psi_i(x): U \rightarrow \mathbb{R}\mid i=1,\ldots,m \}$$ such that all of them are differentiable and ...
NicAG's user avatar
  • 247
0 votes
0 answers
46 views

A direct proof for non-zero limit points of weighted backward shifts

Fix a sequence $(w_1,w_2,\ldots)$ of positive reals such that the linear operator $T: \ell_2\to \ell_2$ given by $$ T(x_1,x_2,x_3,....)=(w_2x_2,w_3x_3,\ldots) \text{ for all sequences in } \ell_2 $$ ...
Paolo Leonetti's user avatar

1
2
3 4 5
48